0 Daumen
333 Aufrufe

Aufgabe:


Angenommen a1,...,an > 0 sind positive reelle Zahlen mit Mittelwert


\(  m = \frac{1}{n} \sum_{i=1}^{n} a_i \)

Zeigen Sie, daß für jede reelle Zahl t > 1 gilt

\( | \{i \in \{1,...,n\}: a_i \geq tm\}| \leq \frac{n}{t} \)

Anmerkung: diese Ungleichung ist als “Markov-Ungleichung” bekannt.



Problem/Ansatz:

ich weiß nicht wirklich wie man hier vorgehen könnte und würde mich über jede Hilfestellung freuen.

Avatar von

1 Antwort

0 Daumen
Avatar von 13 k

Ein anderes Problem?

Stell deine Frage

Willkommen bei der Mathelounge! Stell deine Frage einfach und kostenlos

x
Made by a lovely community